You are on page 1of 74

Test of English Proficiency | 1

2 | Test of English Proficiency


TEST SUMMARY

 The listening test takes approximately 35 minutes.


 Numbers of the questions are 50 multiple-choice questions.
 Listen carefully because the recording will play one time only and the material on the
recording is not written in the test book.
 There are three separate parts. You hear each section once only.
- Part A is Short Dialogues
This part consists of 30 short dialogues and 30 questions about them. Each dialogue is
two-line conversations between two speakers, and then followed by a multiple-choice
question.
- Part B is Long Conversations
This part consists of 2 long conversations (for about 60-90 second) on a casual topic
between students, each followed by a number of multiple-choice questions (usually 4-
5 questions).
- Part C is Talks
This part consists of 3 talks (for about 60-90 seconds) about school life or on
academic subjects, each followed by a number of multiple-choice questions (usually 4
questions).

STRATEGIES FOR LISTENING COMPERENSION


ON PAPER TOEFL TEST

The General strategies

1. Be familiar with the directions.


2. Listen carefully to the passage.
3. Know where the easier and the more difficult questions are generally found.
4. Be familiar with the pacing of the test.
5. Never leave any answers blank on your answer sheet. Guess it.
6. Use any remaining time to look ahead at the answers to the questions that follow.

The Specific strategies

PART A: Focus on the second line of the conversation.

PART B and C: Listen carefully to the first line of the conversation.

While listening to the conversation, follow along with the answer in the test book and try
to determine the correct answers.

Test of English Proficiency | 1


The Language skills needed to answer the questions

SKILL 1: Restatements

Often the correct answer is an answer that contains a restatement of the ideas in the last line of
the dialogue.

Example

On the recording, you hear:

(woman) : Steve, is something the matter? You don’t look very good.
(man) : Oh, I’m feeling a little sick today.
(narrator) : What does the man mean?

In your test book, you read:

(A) He’s not very good-looking.


(B) He’s a bit ill.
(C) He looks worse than he feels.
(D) His feet are a little thick.

In this conversation, sick means ill, and a little means a bit. The best answer to this question is
therefore answer (B) because answer (B) restates the idea in the second line of the dialogue.
Please note that answer (D) is definitely not a correct answer because it contains feet instead of
feel and thick instead of sick. These words are similar in sound but not in meaning to the words
that you hear on the recording.

The Best Strategy: Choose Answers with Restatements

 As you listen to the second line of the conversation, you should focus on the key idea(s)
in that line.
 If you see a restatement of the key idea(s) in a particular answer, then you have probably
found the correct answer.
 Do not choose answers with words that sound similar to the words on the recording.

EXERCISE 1: In this exercise, underline the key idea(s) in the second line of each short
conversation. Then underline restatements of these key words in the answers, and choose the
best answer to each question. Remember that the best answer is probably the answer that
contains a restatement of the key idea(s) in the second line of the conversation.

2 | Test of English Proficiency


1. (woman) : What do you like about your new house?
(man) : It’s very close to a park.
(narrator): What does the man mean?

(A).The house is closed up now.


(B) He parks his car close to his house.
(C) His home is near a park.
(D) He doesn’t really like his new house.

2. (man) : Mark said some really nice things to me.


(woman) : He’s very grateful for what you did.
(narrator): What does the woman say about Mark?

(A) He did a great job.


(B) He bought a crate full of fruit.
(C) He made a great fool of himself. He’s thankful.
(D) He’s thankful.

3. (woman) : Sam, you’ve been working at the computer for hours.


(man) : Yes, and I need to take a break.
(narrator): What does the man mean?

(A) The computer’s broken.


(B) He needs to relax for a while.
(C) He should keep working.
(D) He’s broke.

SKILL 2: Negatives

Negative expressions are very common questions. The common kind of correct response to a
negative statement is a positive statement containing a word with an opposite meaning.

Example

On the recording, you hear:

(woman) : Did you get a lot of work done at the library today?
(man) : I couldn’t. It wasn’t very quiet there.
(narrator) : What does the man mean?

Test of English Proficiency | 3


In your test book, you read:

(A) The library was noisy


(B) He got a lot done.
(C) He couldn’t quite get to the library today.
(D) The library’s a good place to work because it’s quiet.

The correct answer is answer (A). If it was not quiet in the library, this means that it was
noisy. Notice that the correct answer uses noisy, the opposite of quiet.

EXERCISE 2: In this exercise, underline the negative expression in the second line of each
short conversation. Then read the question, and choose the best answer to that question.
Remember that the best answer is one that uses an opposite meaning.

1. (woman) : You made so many mistakes in this homework.


(man) : I wasn’t very careful.
(narrator): What does the man mean?

(A) He was rather careless.


(B) He does not care about mistakes.
(C) He took care of the work at home.
(D) He did not carry the work home.

2. (man) : Is there a lot of soup? I’m kind of hungry.


(woman) : Sorry, there’s not a lot.
(narrator): What does the woman mean?

(A) There’s not very much soap.


(B) She doesn’t like soup.
(C) There’s only a little soup.
(D) The man should not be hungry.

3. (woman) : How was the weather on your trip?


(man) : There wasn’t a cloud in the sky.
(narrator): What does the man mean?

(A) It was cloudy.


(B) He couldn’t see the sky
(C) It wasn’t very crowded.
(D) There was a lot of sunshine.

4 | Test of English Proficiency


SKILL 3: Suggestions

Suggestions are common in the questions, so be familiar with the expressions of suggestions.

Why … not …? Why not …? Let’s …

Example

On the recording, you hear:

(man) : I haven’t talked with my parents in a while.


(woman) : Why don’t you call them now?
(narrator) : What does the woman suggest?

In your test book, you read:

(A) Calling off his visit.


(B) Talking about his parents.
(C) Calling his parents in a while.
(D) Phoning his family.

In this example, the expression why don’t is an expression of suggestion, so the woman’s
suggestion is to call them. This means that the woman suggests phoning his family. The best
answer is therefore answer (D).

EXERCISE 3: In this exercise, underline the expression of suggestion in each


short conversation. Then read the question, and choose the best answer to that question.
Remember that the best answer is one that gives a suggestion.

1. (man) : The weather’s so beautiful today.


(woman) : Let’s walk to school instead of driving.
(narrator) : What does the woman suggest?

(A) Taking the car to school.


(B) Taking a walk instead of going to school.
(C) Going for a drive in the beautiful weather.
(D) Walking to class.

2. (woman) : I never have enough spending money.


(man) : Why not get a part-time job?
(narrator) : What does the man suggest?

Test of English Proficiency | 5


(A) Spending less money.
(B) Doing a better job at work.
(C) Earning some money.
(D) Spending less time at her job.

3. (man) : We don’t have any plans tonight, do we?


(woman) : No, so why don’t we invite some friends over to dinner?
(narrator) : What does the woman suggest?

(A) Having a dinner party.


(B) Stopping the fight with their friends.
(C) Planning a trip.
(D) Making some new friends.

SKILL 4: Passives

If the dialogue contains a passive statement, the answer to the question is often an active
statement, and vice versa.

Check carefully who or what is doing the action in the questions.

Example

On the recording, you hear:

(man) : Is that a new chair?


(woman) : Yes, we just bought it last week.
(narrator) : What does the woman mean?

In your test book, you read:

(A) She brought the chair with her.


(B) The chair was lost for a week.
(C) The chair was purchased recently.
(D) She bought the last chair from the store.

In this conversation, the woman uses an active idea, we. . . bought it [ the chair]. The correct
answer uses the passive idea the chair was purchased. Therefore, the best answer to the question
above is answer (C).

EXERCISE 4: In this exercise, each of the correct answers is either a passive restatement of an
active sentence or an active restatement of a passive sentence. Read the short conversation, and
underline the key active or passive statement. Then read the question, and choose the best answer
to the question. Make sure you understand who or what is doing the action in these passive
sentences.

6 | Test of English Proficiency


1. (man) : What happened to your notebook?
(woman) : I left it in the cafeteria.
(narrator) : What does the woman imply?

(A) The cafeteria is to the left.


(B) She left a note on the cafeteria door.
(C) She took some notes in the cafeteria.
(D) The notebook was left in the cafeteria.

2. (man) : When are we going to talk about the problem with the copy machine?
(woman) : It will be discussed at the meeting tomorrow.
(narrator) : What does the woman mean?

(A) She needs to make copies before the meeting.


(B) They will talk about the problem tomorrow.
(C) It is a problem to have a meeting.
(D) They must discuss whether or not to have a meeting.

3. (woman) : Did you correct the paper?


(man) : I checked every single line again and again.
(narrator) : What does the man mean?

(A) The paper has been thoroughly corrected.


(B) He wrote many of the lines over and over.
(C) The checkmarks were on every line of the paper.
(D) He wrote a check for the paper.

SKILL 5: Who and Where

It is common to draw some kind of conclusion based on clues given in the dialogue. In this type
of question the answer is not clearly stated. Two kinds of conclusion are to determine who the
speaker is (Who is probably talking?), and where the dialogue probably takes place (Where does
the conversation take place?).

Example

On the recording, you hear:

(man) : What do you do during your performances?


(woman) : I play the piano and sing.
(narrator) : Who is the woman most likely to be?

Test of English Proficiency | 7


In your test book, you read:

(A) An athlete.
(B) A member of the audience.
(C) A clerk in a music store.
(D) A musician

The clues performances, piano, and sing in the conversation tell you that the woman is probably
a musician.Answer (D) is therefore the correct answer.

Example

On the recording, you hear:

(man) : I’d like to open an account, please.


(woman) : What type of account would you like, a checking or savings account.
(narrator) : Where does this conversation probably take place?

In your test book, you read:

(A) In an accounting class.


(B) In a bank.
(C) At a service station.
(D) In a market.

The clue checking or savings account in the conversation tells you that
this conversation probably takes place in a bank. Answer (B) is therefore the correct answer.

EXERCISE 5: In this exercise, read the short conversation and question, underline the clues that
help you answer the question, and then choose the best answer. You will have to
draw conclusions about who and where.

1. (man) : I’d like to mail this package, please.


(woman) : First or second class?
(narrator) : Who is the woman most likely to be?

(A) A school administrator.


(B) A postal worker.
(C) A banker.
(D) A teacher.

8 | Test of English Proficiency


2. (woman) : How much are the flowers?
(man) : Ten dollars a bouquet.
(narrator) : Where does this conversation probably take place?

(A) In a bakery
(B) In a grocery store.
(C) In a florist shop.
(D) In a garden.

3. (man) : I need to get this prescription filled.


(woman) : If you’ll take a seat, I’ll have your medicine ready for you in a moment.
(narrator): Who is the woman most likely to be?

(A) A pharmacist.
(B) A baby-sitter.
(C) A flight attendant.
(D) A doctor

SKILL 6: Agreement

Expressions of agreement are common in the dialogues, so be familiar with the expressions.

So do I Me, too. I’ll say You can say that again

Example

On the recording, you hear:

(man) : I thought that the meal was overpriced.


(woman) : Me, too
(narrator) : What does the woman mean?

In your test book, you read:

(A) There were too many spices in the meat.


(B) She has the same opinion of the meal as the man.
(C) She wants to share the man’s meal.
(D) The price of the meal was great.

The expression me, too shows agreement with a positive statement, so the woman means that she
has the same opinion as the man. The best answer is therefore answer (B).

Test of English Proficiency | 9


EXERCISE 6: In this exercise, underline the expression of agreement in each
short conversation. Then read the question, and choose the best answer to that question.
Remember that the best answer is one that shows agreement.

1. (woman) : This homework is going to take forever.


(man) : I’ll say!
(narrator) : What does the man mean?

(A) It’s going to take forever to get home.


(B) It takes a long time to get from home to work.
(C) He and the woman have the same opinion about the homework.
(D) He needs to take the homework to class.

2. (woman) : A trip to the park might be nice,


(man) : You can say that again!
(narrator) : What does the man mean?

(A) The woman should repeat what she said.


(B) It’s nice in the park at night.
(C) The woman should tell him about part of the trip.
(D) He agrees about the trip to the park

3. (man) : I think it’s time to go home.


(woman) : So do I.
(narrator) : What does the woman mean?

(A) They don’t have time to do it.


(B) She also thinks it’s a good idea to leave.
(C) She thinks they are at home.
(D) They are unsure about the time.

10 | Test of English Proficiency


SECTION 1
LISTENING COMPREHENSION
Time-approximately 35 minutes
(including the reading of the directions for each part)
In this section of the test, you will have an opportunity to demonstrate your ability to understand
conversations and talks in English. There are three parts to this section. Answer all the questions
on the basis of what is stated or implied by the speakers you hear. Do not take notes or write in
your test book at any time. Do not turn the pages until you are told to do so.

Part A
Directions: In Part A you will hear short conversations between two people. After each
conversation, you will hear a question about the conversation. The conversation and the
questions will not be repeated. After you hear a question, read the four possible answers in your
test book and choose the best answer. Then on your answer sheet, find the number of question
and fill in the space that corresponds to the letter of the answer you have chosen.
Listen to an example. Sample answer
On the recording you hear:
A B C
(man) That exam was just awful.
(woman) Oh, it could have been worse.
(narrator) What does the woman mean?
In your test book, you read: (A) The exam was really awful.
(B) It was the worst exam she had ever seen.
(C) It couldn’t have been more difficult.
(D) It wasn’t that hard.
You learn from the conversation that the man thought the exam was really difficult and that the
woman disagreed with the man. The best answer to the question, “What does the woman mean?”
is (D), “It wasn’t that hard.” Therefore, the correct choice is (D).

Wait

Test of English Proficiency | 11


1. (A) The coffee is much better this morning.
(B) The coffee tastes extremely good.
(C) The coffee isn’t very good.
(D) This morning he definitely wants some coffee.

2. (A) The two classes meet in an hour and a half.


(B) The class meets three hours per week.
(C) Each half of the class is an hour long.
(D) Two times a week the class meets for an hour.

3. (A) A few minutes ago, the flight departed.


(B) The flight will start in a while.
(C) They are frightened about the departure.
(D) The plane is going to take off soon.

4. (A) He hasn’t yet begun his project.


(B) He’s supposed to do his science project next week.
(C) He needs to start working on changing the due date.
(D) He’s been working steadily on his science project.

5. (A) At the post office


(B) In a florist shop
(C) In a restaurant
(D) In a hospital delivery room

6. (A) The professor drowned the cells in a lab.


(B) The lecture was long and boring
(C) The professor divided the lecture into parts.
(D) The biologist tried to sell the results of the experiment.

7. (A) She needs to get a driver’s license.


(B) Two pieces of identification are necessary.
(C) The man should check to see if he needs credit.
(D) A credit card can be used to get a driver’s license.

8. (A) Housing within his budget is hard to locate.


(B) It’s hard to find his house in New York.
(C) He can’t afford to move his house to New York.
(D) Housing in New York is unavailable.

12 | Test of English Proficiency


9. (A) The boss was working on the reports.
(B) He would have to finish his reports before the end of next month.
(C) He was directed to stay late and finish some work.
(D) He could finish the reports at home.

10. (A) The boisterous students made the teacher mad.


(B) The teacher angered the students with the exam results.
(C) The students were angry that the teacher was around.
(D) The angered students complained to the teacher.

11. (A) The prices are reasonable.


(B) The store is too far out of town.
(C) He would like the woman to repeat what she said.
(D) He agrees with the woman.

12. (A) It’s rained unusually hard this year.


(B) There hasn’t been any rain for many years.
(C) It’s been many years since it rained.
(D) He doesn’t like rain.

13. (A) He needs to do a better job writing questions.


(B) He certainly must make his writing better.
(C) Without the questions, he cannot write the answers.
(D) He needs to understand the written questions better.

14. (A) The agent was standing in line with his passport.
(B) The line to get new passports is very long.
(C) The woman must wait her turn to get her passport checked.
(D) He can check her passport instead of the agent.

15. (A) He couldn’t finish closing the library book.


(B) He hadn’t finished the library assignment, but he was close.
(C) He was working on the assignment when the library closed.
(D) His homework was incomplete because the library wasn’t open.

16. (A) All the lawyer’s preparation did no good.


(B) The lawyer prepared nothing for the case.
(C) It wasn’t work for the lawyer to prepare for the case.
(D) The lawyer didn’t work to prepare for the case.

Test of English Proficiency | 13


17. (A) The history class begins next week.
(B) He thinks the papers should be turned in next week.
(C) He has already done the paper for next week.
(D) The papers are not due next week.

18. (A) He’s not really happy.


(B) The contractor’s work was satisfactory.
(C) He would rather work with the contractor himself.
(D) He was already contacted about the work.

19. (A) The man should try another type of paper.


(B) The man should locate a typist tomorrow morning.
(C) The man should make a tape in the morning
(D) The man should complete the paper without help.

20. (A) She’d like some pie.


(B) It’s easy to buy it.
(C) The task the man’s working on isn’t difficult.
(D) It’s easier to prepare pie than do what the man is doing.

21. (A) He reported that the time for the budget meeting had been set.
(B) He is always late in submitting his accounting figures.
(C) He never manages to budget his time well.
(D) He is never too late in turning in his reports.

22. (A) The repairs that the mechanic had indicated were already made.
(B) The car is going to need a lot of repairs.
(C) Buying a new car would be quite expensive.
(D) The mechanic extended the repair warranty.

23. (A) Betty wrote the letter as directed.


(B) The directions were given to Betty in a letter.
(C) Betty will follow the instructions later.
(D) Betty work exactly as instructed.

24. (A) Walter had a lack of success with his business.


(B) Walter failed in his business.
(C) Walter’s new company is doing rather well.
(D) Walter hoped to succeed in business.

14 | Test of English Proficiency


25. (A) He should put the organ in the closet.
(B) The closet has already been organized.
(C) He needs to rearrange the closet.
(D) He wishes the closet were closer.

26. (A) She didn’t do the work.


(B) She gave the assignment her best effort.
(C) She finished the assignment even though it was difficult.
(D) She gave the man a signal.

27. (A) She said some terrible things.


(B) She didn’t say anything nice.
(C) She didn’t have any nice things.
(D) She said really wonderful things.

28. (A) New employees are rarely initiated into the company.
(B) New workers don’t generally undertake actions on their own.
(C) New employees are initially rated.
(D) It’s rare for employees to make new suggestions.

29. (A) The woman is more than a week late.


(B) The children would have wrecked the house later.
(C) The woman was so late that she was a wreck.
(D) He’s glad that she was not any later.

30. (A) He had not gone to the store


(B) He was still at the market.
(C) He was going to take care of the shopping.
(D) He always went to the market.

Part B

Test of English Proficiency | 15


Directions: In this part of the test, you will hear longer conversations. After each conversation,
you will hear several questions. The conversations and questions will not be repeated.
After you hear a question, read the four possible answers in your test book and choose the best
answer. Then, on your answer sheet, find the number of the question and feel in the space that
corresponds to the letter of the answer you have chosen.
Remember, you are not allowed to take notes or write in your test book.

31. (A) She’s a senior.


(B) She’s a junior.
(C) She’s a transfer student.
(D) She’s a graduate student.

32. (A) How to transfer to a junior college.


(B) How to find his way around campus.
(C) What courses are required for a literature major
(D) Who won the campus election

33. (A) Three


(B) Five
(C) Eight
(D) Ten

34. (A) American literature


(B) World literature
(C) Literacy analysis
(D) Surveying

35. (A) In a book.


(B) From a television program.
(C) During a trip that she took.
(D) From a lecture.

36. (A) To communicate with other dolphins


(B) To recognize objects in the water
(C) To learn human language
(D) To express fear

16 | Test of English Proficiency


37. (A) Five
(B) Fifteen
(C) Fifty
(D) Five hundred

38. (A) It is limited.


(B) It is greater than human intelligence.
(C) It is less than previously thought.
(D) We are beginning to learn how much they have.

Test of English Proficiency | 17


Part C
Directions: In this part of the test you will hear several talks. After each talk, you will hear some
questions. The talks and the questions will not be repeated.
After you hear a question, read the four possible answers in your test book and choose the best
answer. Then, on your answer sheet, find the number of the question and fill in the space that
corresponds to the letter of the answer you have chosen.
Here is an example.
On the recording you hear:
(narrator) Listen to an instructor talk to his class about painting.
(man) Artist Grant Wood was a guiding force in the school of painting known as
American regionalist, a style reflecting the distinctive characteristics of art from rural areas of
the United States. Wood began drawing animals on the family farm at the age of three, and when
he was thirty-eight one of his paintings received a remarkable amount of public notice and
acclaim. This painting, called “American Gothic,” is a starkly simple depiction of a serious
couple staring directly out at the viewer.

Now, listen to a sample question.

(narrator) What style of painting is known as American regionalist?

In your test book, you read: (A) Art from America’s inner cities
(B) Art from the central region of the United States
(C) Art from various urban areas in the United States.
(D) Art from rural sections of America
The best answer to the question, “What style of painting is known as American regionalist?” is
(D), “Art from rural sections of America.” Therefore, the correct choice is (D).

Now, listen to another sample question.

(narrator) What is the name of Wood’s most successful painting?

In your test book, you read: (A) “American Regionalist”


(B) “The family farm in Iowa”
(C) “American Gothic”
(D) “A serious couple”
The best answer to the question, “What is the name of Wood’s most successful panting?” is (C),
“American Gothic.” Therefore, the correct choice is (C).
Remember, you are not allowed to take notes or write in your test book.

18 | Test of English Proficiency


39. (A) To protect its member
(B) To save the natural environment
(C) To honor the memory of John Muir
(D) To improve San Fransisco’s natural beauty

40. (A) For less than a year


(B) Only for a decade
(C) For more than a century
(D) For at least two centuries

41. (A) San Fransisco


(B) All fifty states
(C) The Sierra Nevadas
(D) The eastern United States

42. (A) All over the world


(B) In the entire United States
(C) Only in California
(D) Only in the Sierra Nevadas

43. (A) Students signing up for athletic teams


(B) Students going on a tour of a university campus
(C) Students playing various sports
(D) Students attending a university dedication ceremony

44. (A) Membership on an athletic team


(B) Enrollment in an exercise class
(C) A valid student identification card
(D) Permission from a faculty member

45. (A) To the tennis courts


(B) To the arena
(C) To the gymnasium
(D) To the Athletic Department office

46. (A) Go to the Art Center


(B) Sign up for sports classes
(C) Visit the exercise room
(D) Watch a football game

47. (A) Science


(B) Art
(C) Literature
(D) Music

Test of English Proficiency | 19


48. (A) They are completely different.
(B) They are somewhat similar but have an essential difference.
(C) They are exactly the same in all respects.
(D) They are unrelated.

49. (A) Objective


(B) Idealistic
(C) Philosophical
(D) Environmental

50. (A) Heredity


(B) Environment
(C) Idealism
(D) Natural laws

20 | Test of English Proficiency


STRUCTURE

SKILL 1: BE SURE THE SENTENCE HAS A SUBJECT AND A VERB

You know that a sentence in English should have a subject and a verb. The most common types
of problems that you will encounter in structure questions on the TOEFL test have to do with
subjects and verbs: perhaps the sentence is missing either the subject or the verb or both, or
perhaps the sentence has an extra subject or verb.

The following chart outlines what you should remember about subjects and verbs:

SUBJECTS AND VERBS


A sentence in English must have at least one subject and one verb.

SKILL 2: BE CAREFUL OF PRESENT PARTICIPLES

A present participle is the –ing form of the verb (talking, playing). In structure questions on the
TOEFL test, a present participle can cause confusion because it can be either a part of the verb or
an adjective. It is part of the verb when it is preceded by some form of the verb be.

The man is calling to his friend.


VERB

In this sentence talking is part of the verb because it is accompanied by is.


A present participle is an adjective when it is not accompanied by some form of the verb
be.
The man talking to his friend has a beard.
ADJECTIVE

The following chart outlines what you should remember about present participles:
PRESENT PARTICIPLES
A present participle is the –ing form of the verb. The present participle can be (1) part of the
verb (2) an adjective. It is part of the verb when it is accompanied by some form of the verb be.
It is an adjective when it is not accompanied by some form of the verb be.
1. The boy is standing in the corner.
2. The boy standing in the corner.

Test of English Proficiency | 21


SKILL 3: BE CAREFUL OF PAST PARTICIPLES

Past participle can cause confusion on the TOEFL test because a past participle can be either an
adjective or a part of a verb. The past participle is the form of the verb that appears with have or
be. It often ends in –ed, but there are also many irregular participles in English.
The family has purchased a television.
VERB

The poem was written by Paul.


VERB

In the first sentence the past participle purchased is part of the verb because it is accompanied by
has. In the second sentence the past participle written is part of the verb because it is
accompanied by was.
A past participle is an adjective when it is not accompanied by some form of be or have.

The television purchased yesterday was expensive.


ADJECTIVE

The poem written by Paul appeared in the magazine.


ADJECTIVE

In the first sentence purchased is an adjective rather than a verb because it is not accompanied by
a form of be or have (and there is verb, was, later in the sentence). In the second sentence written
is an adjective rather than a verb because it is accompanied by a form of be or have (and there is
a verb, appeared, later in the sentence).
The following chart outlines what you should remember about past participles:

PAST PARTICIPLE
A past participle often ends in -ed, but there are also many irregular past participles. For many
verbs, including -ed verbs, the simple past and the past participle are the same and can be easily
confused. The -ed form of verb can be (1) the simple past

Exercise skill 1-3

1. The North Platte River from Wyoming into Nebraska.


(A) it flowed
(B) flows
(C) flowing
(D) with flowing waters

22 | Test of English Proficiency


2. A pride of lions up to forty lions, including one to three males, several females,
and cubs.
(A) can contain
(B) it contains
(C) contain
(D) containing
3. Any possible academic assistance from taking stimulants marginal at best.
(A) it is
(B) there is
(C) is
(D) as
4. Henry Adams, born in Boston, famous as a historian and novelist.
(A) became
(B) and became
(C) he was
(D) and he became
5. A computerized map of the freeways using information gathered by sensors embedded in
the pavement on a local cable channel during rush hours.
(A) airs
(B) airing
(C) air
(D) to air

SKILL 4: USE COORDINATE CONNECTORS CORRECTLY

When you have two clauses in an English sentence, you must connect the two clauses correctly.
One way to connect two clauses is to use and, but, or, so, or yet between the clauses.

Tom is singing, and Paul is dancing.

Tom is tall, but Paul is short.

Tom must write the letter, or Paul will do it.

Tom told a joke, so Paul laughed

Tom is tired, yet he is not going to sleep.

Test of English Proficiency | 23


The following chart lists the coordinate connectors and the sentence pattern used with them:

COORDINATE CONNECTORS
and but or so yet
S V S V
coordinate connector
She laughed, or she wanted to cry.

SKILL 5: USE ADVERB TIME AND CAUSE CONNECTORS CORRECTLY

Sentences with adverb clause have two basic patterns in English. Study the clauses and
connectors in the following sentences:

I will sign the check before you leave.

Before you leave, I will sign the check.

In each pf these examples, there are two clauses; you leave and I will sign the check, and the
clause you leave is an adverb time clause because it is introduced with the connector before. In
the first example the connector before comes in the middle of the sentence, and no comma (,) is
used. In the second example the connector comes at the beginning of the sentence, a comma (,) is
required in the middle of the sentence.
The following chart lists adverb time and cause connectors and the sentence patterns used with
them:

ADVERB TIME AND CAUSE CONNECTORS


TIME CAUSE
after as soon as once when as now that
as before since whenever because since
as long as by the time until while in as much as
S V S V
adverb connector

Teresa went inside because it was raining.

adverb connector
S V, S V

Because it was raining, Teresa went inside.

24 | Test of English Proficiency


SKILL 6: USE OTHER ADVERB CONNECTORS CORRECTLY

Adverb clause can express the ideas of time and cause, as you saw in Skill 7; adverb clauses can
also express a number of other ideas, such as contrast, condition, manner, and place. Because
these clauses are adverb clauses, they have the same structure as the time and cause clauses in
Skill 7. Study the following examples:

I will leave at 7:00 if I am ready.

Although I was late, I managed to catch the train.

In each of these examples, there are two clauses that are correctly joined with adverb connectors.
In the first sentence, the adverb condition connector if comes in the middle of the sentence. In the
second sentence, the adverb contrast connector although comes at the beginning of the sentence,
and a comma (,) is used in the middle of the sentence.
The following chart lists adverb contrast, condition manner, and place connectors and the
sentence patterns used with them.
OTHER ADVERB CONNECTORS
CONDITION CONTRAST MANNER PLACE
if although as where
in case even though in that wherever
provided though
providing
while
unless
whether whereas
S V S V
adverb connector
Bob went to school eventhough he felt sick.

S V, S V
adverb connector

Eventhough Bob felt sick, he went to school.

NOTE: A comma is often used in the middle of the sentence with a contrast connector.
The Smith family arrived at 2:00, while the Jones family arrived an hour later.

Test of English Proficiency | 25


Exercise skill 4-6

1. The president of the United States appoints the cabinet members, appointments
are subject to Senate approval.
(A) their
(B) with their
(C) because their
(D) but their
2. The prisoners were prevented from speaking to reporters because .
(A) not wanting the story in the papers
(B) the story in the papers the superintended did not want
(C) the public to hear the story
(D) the superintended did not want the story in the papers

3. case studies arethe target of much skepticism in the scentific community, used
extensively by numerous researchers..
(A) they are
(B) are
(C) yet they
(D) yet they are
4. sucked groundwater from below, some parts of the city have begun to sink as
much as ten inches annually.
(A) Pumps have
(B) As pumps have
(C) So pumps have
(D) With pumps
5. According to the hypothesis in the study, the monarchs pick up the magnetic field of the
. migrate by following magnetic fields.
(A) target monarchs
(B) target since monarchs
(C) target since monarchs are
(D) target

26 | Test of English Proficiency


SKILL 7: USE NOUN CLAUSE CONNECTORS CORRECTLY

A noun clause is a clause that functions as a noun; because the noun clause is a noun, it is used in
a sentence as either an object of a verb, an object of preposition, or the subject of the sentence.

I know when he will arrive.


NOUN CLAUSE AS OBJECT OF VERB

I am concerned about when he will arrive.


NOUN CLAUSE AS OBJECT OF PREPOSITION

When he will arrive is not important.


NOUN CLAUSE AS SUBJECT

In the first example there are two clauses, I know and he will arrive. These two clauses are joined
with the connector when. When changes the clause he will arrive into a noun clause that
functions as the object of the verb know.
In the second example the two clauses I am concerned and he will arrive are also joined
by the connector when. When changes the clause he will arrive into a noun clause that functions
as the object of preposition about.
The third example is more difficult. In this example there are two clauses, but they are a
little harder to recognize. He will arrive is one of the clauses, and the connector when changes it
into a noun clause that functions as the subject of the sentence. The other clause has the noun
clause when he will arrive as its subject and is as its verb.
The following chart lists the noun clause connectors and the sentence patterns used with
them.
NOUN CLAUSE CONNECTORS
 What, when, where, why, how  Whatever, whenever  Whether, if  that

NOUN CLAUSE AS OBJECT

S V noun connector S V
I know what you did.

NOUN CLAUSE AS SUBJECT

noun connector S V V
What you did was wrong.

Test of English Proficiency | 27


SKILL 8: USE NOUN CLAUSE CONNECTOR/SUBJECTS CORRECTLY

In skill 9 we saw that noun clause connectors were used to introduce noun subject clauses or
noun object clauses. In skill 10 we will see that in some cases a noun clause connector is not just
a connector; a noun clause connector can also be the subject of the clause at the same time.

I do not know what is in the box.


NOUN CLAUSE AS OBJECT OF VERB

We are concerned about who will do the work.

NOUN CLAUSE AS OBJECT OF PREPOSITION

Whoever is coming to the party must bring a gift.


NOUN CLAUSE AS SUBJECT

In the first example there are two clauses. I do not know and what is in the box. These
two clauses are joined by the connector what. It is important to understand that in this sentence
the word what serves two functions. It is both the subject of the verb is and the connector that
joins the two clauses.
In the second example there are two clauses. In the first clause we is the subject of are. In
the second clause who is the subject of will do. Who also serves as the connector that joins the
two clauses. The noun clause who will do the work functions as the object of the preposition
about.
In the last example there are also two clauses. Whoever is the subject of the verb is
coming, and the noun clause whoever is coming to the party is the subject of must bring. The
word whoever serves two functions in the sentence: It is the subject of the verb is coming, and it
is the connector that joins two clauses.
The following chart lists the noun clause connector/subjects and the sentence patterns
used with them:

28 | Test of English Proficiency


NOUN CLAUSE CONNECTOR/SUBJECTS
who what which
whoever whatever whichever

NOUN CLAUSE AS OBJECT

S V noun connector V
I know what happened.

NOUN CLAUSE AS SUBJECT

noun connector
V V
What happened was great.

SKILL 9: USE ADJECTIVE CLAUSE CONNECTORS CORRECTLY

An adjective clause is a clause that describes a noun. Because the clause is an adjective, it is
positioned directly after the noun that is describes.

The woman is filling the glass that she put on the table.
ADJECTIVE CLAUSE

The glass that she put on the table contains milk.

ADJECTIVE CLAUSE

In the first example there are two clauses: woman is the subject of the verb is filling, and
she is the subject of the verb put. That is the adjective clause connector that joins these two
clauses, and the adjective clause that she put on the table describes the noun glass.
In the second example there are also two clauses: glass is the subject of the verb contains,
and she is the subject of the verb put. In this sentence also, that is the adjective clause connector
that joins these two clauses, and the adjective clause that she put on the table describes the noun
glass.
The following chart lists the adjective clause connectors and the sentence patterns used
with them:

Test of English Proficiency | 29


ADJECTIVE CLAUSE CONNECTORS
whom which that
(for people) (for things) (for people or things)

S V adjective connector S V
I liked the book which you recommended.

S adjective connector S V V
The book which you recommended was interesting.
NOTE: The adjective connectors can be omitted. This omission is very common in spoken
English or in casual written English. It is not as common in formal English or in structure
questions on the TOEFL test.

SKILL 10: USE ADJECTIVE CLAUSE CONNECTORS/SUBJECTS CORRECTLY

In skill 11 we saw that adjective clause connectors were used to introduce clause that describes
nouns. In skill 12 we will see that in some cases an adjective clause connector is not just a
connector; an adjective clause connector can also be the subject of the clause at the same time.

The woman is filling the glass that is on the table.

ADJECTIVE CLAUSE

The glass that is on the table contains milk.


ADJECTIVE CLAUSE

In the first example there are two clauses: woman is the subject of verb is filling, and that
is the subject of verb is. These two clauses are joined with the connector that. Notice that in this
example the word that serves as two functions at the same time: it is the subject of the verb is,
and it is the adjective clause connector that joins the two clauses. The adjective clause that is on
table describes the noun glass.

30 | Test of English Proficiency


In the second example, there are also two clauses: glass is the subject of verb contains,
and that is the subject of verb is. In this example that also serves as two functions at the same
time: it is the subject of the verb is, and it is the adjective clause connector that joins the two
clauses. Because that is on the table describes the noun glass, it directly follows glass.
The following chart lists the adjective clause connector/subjects and the sentence patterns
used with them:
ADJECTIVE CLAUSE CONNECTOR/SUBJECTS
whom which that
(for people) (for things) (for people or things)

S V adjective connector V
She needs a secretary who types fast.

S adjective connector V V
A secretary who types fast is invaluable.

Exercise skill 7-10

1. Dolphins from extremely complicated alleguances and continually change.


(A) enmities that
(B) that are enmities
(C) enmities that are
(D) that enmities

2. During free fall, up to a full minute, a skydiver will fall at a constant speed of
120 m.p.h.
(A) it is
(B) which is
(C) being
(D) is
3. will be carried in the next space shuttle playload has not yet been announced to
the public.
(A) It
(B) What
(C) When
(D) That

Test of English Proficiency | 31


SKILL 11: INVERT THE SUBJECT AND VERB WITH NEGATIVES

The subject and verb can also be inverted after certain negatives and related expressions such as
no, not, or never, come at the beginning of a sentence, the subject and verb are inverted.
Not once did I miss a question.

Never has Mr. Jones taken a vacation.

At no time can the woman talk on the telephone.

In the first example the negative expression not once causes the subject I to com after the helping
verb did. In the second example the negative word never causes the subject Mr. Jones to come
after helping verb has. In the last example the negative expression at no time causes the subject
woman to come after the helping verb can.

Certain word in English, such as hardly, barely, scarcely, and only, act like negatives. If
one of these words comes at the beginning of a sentence, the subject and verb are also inverted.

Hardly ever does he take time off.


(This means that he almost never takes time off.)

Only once did the manager issue overtime paychecks.


(This means that the manager almost never issued overtime paychecks.)

In the first example the “almost negative” expression hardly ever causes the subject he to come
after the helping verb does. In the second example the “almost negative” expression only once
causes the subject manager to come after the helping verb did.

When a negative expression appears in front of a subject and verb in the middle of a
sentence, the subject and verb are also inverted. This happens often with the negative words
neither and nor.

I do not want to go, and neither does Tom.

The secretary is not attending the meeting, nor is her boss.

In the first example the negative neither causes the subject Tom to come after the helping verb
does. In the second example the negative nor causes the subject boss to come after the verb is.

32 | Test of English Proficiency


The following chart is the negative expressions and the sentence patterns used with them:

INVERTED SUBJECTS AND VERBS WITH NEGATIVES


No not never neither nor
Barely hardly only rarely scarcely seldom

When a negative expression appears in front of a subject and verb (at the beginning of a sentence
or in the middle of a sentence) the subject and verb are inverted.
negative expression
V S
Rarely were they so happy.

SKILL 12: INVERT THE SUBJECT AND VERB WITH CONDITIONALS

In certain conditional structures, the subject and verb may also be inverted. This can occur when
the helping verb in the conditional clause is had, should, or were and the conditional connector if
is omitted.

If he had taken more time, the results would have been better.

Had he taken more time, the results would have been better.

I would help you if I were in a position to help.

I would help you were I in a position to help.

SKILL 13: INVERT THE SUBJECT AND VERB WITH PLACE EXPRESSIONS

After ideas expressing place, the subject and the verb sometimes invert in English. This can
happen with single words expressing place such as here, there, or nowhere.

Here is the book that you lent me.

There are the keys that I thought I lost.

Nowhere have I seen such beautiful weather.

Exercise skill 11-13

1. Rarely located near city lights or at lower elevations.


(A) observatories are
(B) are
(C) in the observatories
(D) are observatories

Test of English Proficiency | 33


2. Nowhere more skewed than in the auto industry.
(A) that retail trade figures
(B) retail trade figures are
(C) are retail trade figures
(D) retail trade figures
3. Were millions of dollars each year replenishing eroding beaches, the coastline
would be changing even more rapidly.
(A) the U.S. Army Corps of Engineers not spending
(B) the U.S. Army Corps of Engineers not spend
(C) the U.S. Army Corps of Engineers does not spending
(D) not spending the U.S. Army Corps of Engineers

SKILL 14: MAKE VERBS AGREE AFTER EXPRESSIONS OF QUANTITY

A particular agreement problem occurs when the subject is an expression of quantity, such as all,
most or some, followed by the preposition of. In this situation, the subject (all, most, or some) can
be singular or plural, depending on what follows the preposition of.

All (of the book) was interesting.


SINGULAR

All (of the books) were interesting.


PLURAL

All (of the information) was interesting.


UNCOUNTABLE

In the first example the subject all refers to the singular noun book, so the correct verb is
therefore the singular verb was. In the second example the subject all refers to noun books, so the
correct verb is the plural verb were. In the third example the subject all refers to the uncountable
noun information, so the correct verb is therefore the singular verb was.
The following chart outlines the key information that you should understand about
subject/verb agreement after expressions of quantity:

SUBJECT/VERB AGREEMENT AFTER EXPRESSIONS OF QUANTITY


all
most
some OF THE (OBJECT)
half
When an expression of quantity is the subject, the verb agrees with the object.

34 | Test of English Proficiency


SKILL 15: MAKE VERBS AGREE AFTER CERTAIN WORDS

SUBJECT/VERB AFTER CERTAIN WORDS


These words are always singular, so they take singular verbs.
anybody everybody nobody somebody each
anyone everyone anybody someone every
anything everything anything something

Exercise skill 14

1. Among bees a highly elaborate form of communication.


(A) occur
(B) occurs
(C) it occurs
(D) they occur
2. heated by solar energy have special collectors on the roofs to trap sunlight.
(A) A home is
(B) Homes are
(C) A home
(D) Homes

3. Each number in a binary system are formed from only two symbols.
A B C D

4. Scientists at the medical center is trying to determine if there is a relationship between


A B C
saccharine and cancer.
D

SKILL 16: USE PARALLEL STRUCTURE WITH COORDINATE CONJUNCTIONS

The job of the coordinate conjunctions (and, but, or) is to join together equal expressions. In
other words, what is on one side of these words must be parallel to what is on the other side.
These conjunctions can join nouns, or verbs, or adjectives, or phrases, or subordinate clauses, or
main clauses; they just must join together two of the same thing. Here are examples of two nouns
joined by a coordinate conjunction:

I need to talk to the manager or the assistant manager.


She is not a teacher but a lawyer.
You can choose from activities such as hiking and kayaking.

Test of English Proficiency | 35


Here are examples of two verbs joined by a coordinate conjunction:
He only eats and sleeps when he takes a vacation.
She invites us to her home but never talks with us.
You can stay at home or go to the movies with us.
Here are examples of two adjectives joined by a coordinate conjunction:
My boss is sincere and nice.
The exam that he gave was short but difficult.
Class can be interesting or boring.
Here are examples of two phrases joined by a coordinate conjunction:
There a students in the classroom and in front of the building.
The papers are on my desk or in the drawer.
The checks will be ready not at noon but at 1:00
Here are examples of two clauses joined by a coordinate conjunction:
They are not interested in what you say or what you do.
I am here because I have to be and because I want to be.
Mr. Brown likes to go home early, but his wife prefers to stay late.
The following chart outlines the use parallel structures with coordinate conjunctions:

PARALLEL STRUCTURE WITH COORDINATE CONJUNCTIONS


and
(same structure) but (same structure)
Or
and
(same structure), (same structure), but (same structure)
Or

SKILL 17: USE PARALLEL STRUCTURE WITH PAIRED CONJUNCTIONS

The paired conjunctions both… and, either… or, neither… nor, and not only… but also rewuie
parallel structures.
I know both where you went and what you did.
Either Mark or Sue has the book.
The tickets are neither in my pocket nor in my purse.
He is not only an excellent student but also an outstanding athlete.
When you are using these paired conjunctions, be sure that the correct parts are used together.

36 | Test of English Proficiency


The following chart outlines the use of parallel structure with paired conjunctions:

PARALLEL STRUCTURE WITH PAIRED CONJUNCTIONS

both and
either or
(same structure) (same structure)
neither nor
not only but also

SKILL 18: USE PARALLEL STRUCTURE WITH COMPARISONS

When you make a comparison, you point out the similarities or differences between two things,
and those similarities or differences must be in parallel form. You can recognize a comparison
showing how two things are different from the –er…than or more…than.

My school is farther than your school.


To be rich is better than to be poor.
What is written is more easily understood than what is spoken.
A comparison showing how tw things are the same might contain as….as or expressions such as
the same as or similar to.
Their car is as big as a small house.
Renting those apartments costs about the same as leasing them.
The work that I did is similar to the work that you did.

The following chart outlines the use of parallel structure with comparisons:
PARALLEL STRUCTURE WITH COMPARISONS

more . . . than
-er . . . than
less . . . than
(same structure) as . . . as (same structure)
the same . . . as
similar . . . to

Test of English Proficiency | 37


Exercise skill 16-18

1. Truman Capote’s In Cold Blood is neither journalistically accurate .


(A) A piece of fiction
(B) Nor a fictitious work
(C) Or written in a fictitious way
(D) Nor completely fictitious
2. vitamin C is necessary for the prevention and of scurvy.
(A) It cures
(B) cures
(C) cure
(D) for curing
3. A baby’s development in influenced by both heredity and .
(A) By environmental factors
(B) Environmentally
(C) The influence of the environment
(D) Environment

4. Fire extinguishers can contain liquefied gas, dry chemicals, or watery.


A B C D
5. The U.S. Congress consists of both the Senate as well as the House of Representatives.
A B C D
6. The prison population in this state, now at an all time high, is higher than any state.
A B C D

SKILL 19: FORM COMPARATIVES AND SUPERLATIVES CORRECTLY

The comparative is formed with either –er or more and than. In the comparative, -er is used with
short adjectives such as tall, and more is used with longer adjectives such as beautiful.
Bob is taller than Ron.
Sally is more beautiful than Sharon.
The superlative is formed with the, either –est or most, and sometimes in, of, or a that-clause. In
the superlative, -est is used with short adjectives such as tall, and most is used with longer
adjectives such as beautiful.
Bob is the tallest man in the room.
Sally is the most beautiful of all women at the party.
The sider over there is the largest one that I have ever seen.
The fastest runner wins the race. (no in, of, or that)

38 | Test of English Proficiency


The following chart outlines the possible forms of the comparatives and superlatives:

THE FORM OF COMPARATIVES AND SUPERLATIVES

more (long adjective)


COMPARATIVE than
(short adjective) + er

most (long adjective)


SUPERLATIVE than maybe in, of, that
(short adjective) + est

SKILL 20: USE COMPARATIVES AND SUPELATIVES CORRECTLY

The comparative and superlative have different uses, and it is important to understand these
different uses. The comparative is used to compare two equal things.
The history class is larger than the math class.
Mary is more intelligent than Sue.
In the first example the history class is being compared with the math class. In the second
example Mary is being compared with Sue.
The superlative is used when there are more than two items to compare and you want to
show the one that is the best, the biggest, or in some way the most outstanding.
The history class is the largest in the school.
Mary is the most intelligent of all the students in the class.
In the first example the history class s compared all the other classes in the school, and the
history class is larger than each of the other classes. In the second example, Mary is compared
with all the other students in the class, and Mary is more intelligent than each of the other
students.

The following chart outlines the uses of comparatives and superlatives:

THE USE OF COMPARATIVES AND SUPERLATIVES


The COMPARATIVE is used to compare two equal things.
The SUPERLATIVE is used to show which one of many is in some way the most outstanding.

Test of English Proficiency | 39


SKILL 21: USE THE IRREGULAR –ER, -ER STRUCTURE CORRECTLY

An irregular comparative structure that has been appearing frequently on the TOEFL test
consists of two parallel comparatives introduced by the.
The harder he tried, the further he fell behind.
The older the children are, the more their parents expect from them.
The first example contains the two parallel comparatives, the harder and the further. The second
example contains the two parallel comparatives, the older and the more.
In this type of sentence, the and the comparison can be followed by a number of different
structures.
The more children you have, the bigger he house you need.
The harder you work, the more you accomplish.
The greater the experience, the higher the salary.
In the first example, the more is followed by the noun children and the subject and verb you
have, while the bigger is followed by the noun the house and the subject and verb you need. In
the second example, the harder is followed by the subject and verb you work, while the more is
followed by the subject and verb you accomplish. In the third example, the greater is followed
by the noun the experience, while the higher is followed by the noun the salary. You should note
that the last example does not even contain a verb, yet it is a correct structure in English.

The following chart outlines the irregular –er, -er structure:


THE –ER, -ER STRUCTURE

-er -er
THE (the same structure), THE (the same structure)
more more

This type of sentence may or may not include verb.

Exercise skill 19-21

1. The speed of light is the speed of sound.


(A) faster
(B) much faster than
(C) the fastest
(D) as fast
2. The use of detail is method of developing a controlling idea, and almost all
stduents employ this method.
(A) More common
(B) common
(C) most common
(D) the most common

40 | Test of English Proficiency


3. in Stevenson’s landscapes, the more vitality and character the paintings seem to
possess.
(A) The brushwork is loose
(B) The looser brushwork
(C) The loose brushwork is
(D) The loosed the brushwork is

SKILL 22: AFTER WILL, WOULD, OR OTHER MODALS, USE THE BASE FORM OF
THE VERB

Whenever you see a modal, such as will, would, shall, shall, should, can, could, may, might, or
must, you should be sure that the verb that follows it is in its base form.
The boat will leaving* at 3:00 (should be will leave)
The doctor may arrives* soon. (should be may arrive)
The students must taken* the exam (should be must take)

The following chart outlines the use of verb forms after modals:

VERBS FORMS AFTER MODALS


MODAL + base form of the verb

KILL 23: AFTER BE, USE THE PRESENT PARTICIPLE OR THE PAST PARTICIPLE
The verb be in any of its forms (am, is, are, were, was, be, been, being) can be followed by
another verb. This verb should be in the present participle or past participle form.
We are do our homework. (should be are doing)
The homework was do early. (should be was done)
Tom is take the book. (should be is taking)
The book was take by Tom. (should be was taken)

Exercise skill 22-23

1. Alice in Wonderland, first published in 1865, has since being translated into thirty
A B C D
languages.

Test of English Proficiency | 41


2. The Peace Corps was establish on March 1 1961, by then President John F. Kennedy.
A B C D
3. The advisor told himself, while listening to the speech, that a dozen other reporters
A B
would has already asked that question.
C D
4. The deposit will has to be paid before the apartment can be rented.
A B C D

SKILL 24: USE THE CORRECT SINGULAR OR PLURAL NOUN

In written expression questions on the TOEFL test, you should watch very carefully for the
keywords, such as each, every, a, one, and single, that indicate that a noun should be singular.
You should also watch carefully for such keywords as many, several, both, various, and two (or
any other number except one) that indicate that a noun should be plural.
The following chart lists the key words that indicate to you whether a noun should be singular or
plural:
KEYWORDS FOR SINGULAR AND PLURAL NOUNS
For Singular Nouns each every single one a
For Plural Nouns Both two many several various

SKILL 25: DISTINGUISH COUNTABLE AND UNCOUNTABLE NOUNS

In English, nouns are classified as countable or uncountable. For certain questions on the TOEFL
test, it is necessary to distinguish countable and uncountable nouns in order to use the correct
modifiers with them.
As the name implies, countable nouns are nouns hat can be counted. Countable nouns can
come in quantities of one, or two, or a hundred, etc. the noun book is countable because you can
have one book or several books.
Uncountable nouns, on the other hand, are nouns that cannot be counted because they
come in some indeterminate quantity or mass. A noun such as milk or happiness cannot be

42 | Test of English Proficiency


counted; you cannot have one milk or two milks, and you cannot find one happiness or two
happinesses. Uncountable nouns are often liquid items, such as water, oil, or shampoo.
Uncountable nouns can also refer to abstract ideas, such as security, excitement, or hope.
It is important for you to recognize the difference between countable and uncountable
nouns when you come across such keywords as much and many.
He has seen much* foreign films.
He didn’t have many* fun at the movies.
In the first example, much is incorrect because films is countable. This sentence should say many
foreign films. In the second example; many is incorrect because fun is uncountable. This sentence
should say much fun.
The following chart lists the keywords that indicate to you whether a noun should be
countable or uncountable.

KEYWORDS FOR COUNTABLE AND UNCOUNTABLE NOUNS


For Countable Nouns many number few fewer
For Uncountable Nouns much amount little less

SKILL 26: RECOGNIZE IRREGULAR PLURALS OF NOUNS


Many nouns in English have irregular plurals, and these irregular forms can cause confusion in
written expression questions on the TOEFL test. The irregular forms that are the most
problematic are plurals forms that do not end in s.

The following chart lists the irregular plurals that you should become familiar with:

IRREGULAR PLURALS
Vowel change man / men foot / feet goose / geese
woman / women tooth / teeth mouse / mice
Add-EN child / children ox / oxen
Same as deer / deer salmon / salmon trout / trout
singular fish / fish sheep / sheep
analysis / analyses diagnosis / diagnoses synthesis / syntheses

Test of English Proficiency | 43


-is -es axis / axes hypothesis / hypotheses thesis / theses
crisis / crises parenthesis / parentheses
Ends in -A bacterium / bacteria datum / data criterion / criteria
curriculum / curricula phenomenon / phenomena
alumnus / alumni fungus / fungi stimulus / stimuli
-us -i bacillus / bacilli nucleus / nuclei syllabus / syllabi
cactus / cacti radius / radii

Exercise skill 24-26

1. As a compilation of useful details, a weekly magazine commends itself in several respect.


A B C D
2. Through aquaculture, or fish farming, more than 500 million tons of fish are produced
A B C
each years.
D
3. The legal system has much safeguards to protect the right of a defendant to an impartial
A B C
jury.
D
4. Fungus cause more serious plant diseases than do other parasites.
A B C D

SKILL 27: DISTINGUISH POSSESIVE ADJECTIVES AND PRONOUNS

Possessive adjectives and pronouns both show who or what “owns” a noun. However, possessive
adjectives and possessive pronouns do not have the same function, and these two kinds of
possessives can be confused on the TOEFL test. A possessive adjective describes a noun: it
cannot be accompanied by a noun.
They lent me their book.
ADJECTIVE

They lent me theirs.


PRONOUN

Notice that in the first example the possessive adjective there is accompanied by the noun book.
In the second example the possessive pronoun theirs is not accompanied by a noun
44 | Test of English Proficiency
The following chart outlines the possessives and their uses:
POSSESIVE POSSESSIVE
ADJECTIVES PRONOUNS
My mine
your yours
his his
her hers
its -
our ours
their theirs
must be accompanied by a noun. cannot be accompanied by a noun

Skill 28: CHECK PRONOUN REFERENCE FOR AGREEMENT


After you have checked that the subject and object pronouns and the possessiveness for
agreement. The following are examples of errors of this type that you might find on the TOEFL
test:
The boys will cause trouble if you let him*.
Everyone must give their* name.
In the first example, the singular pronoun him is incorrect because it refers to the plural noun
boys. This pronoun should be replaced with the plural pronoun them. In the second example, the
plural possessive adjective their is incorrect because it refers to the singular everyone. This
adjective should be replaced with the singular his or her.
The following chart outlines what you should remember about checking pronoun
reference:
PRONOUN AGREEMENT
1. Be sure that every pronoun and possessive agrees with the noun it refers to.
2. You generally check back in the sentence for agreement.

Exercise skill 27-28

1. Superman made their comic debut in 1938 in Action Comics.


A B C D
2. Commercial letters of credit are often used to finance export trade, but them can have
A B C
other uses.
D

Test of English Proficiency | 45


3. When children experience too much frustration, its behavior ceases to be integrated.
A B C D

SKILL 29: USE BASIC ADJECTIVES AND ADVERBS CORRECTLY

Sometimes in written expression questions on the TOEFL test, adjectives are used in place of
adverbs, or adverbs are used in place of adjectives. Adjectives and adverbs have very different
users. Adjectives have only one job: they describe nouns or pronouns.

She is a beautiful woman.


ADJ. NOUN

She is beautiful.
PRO ADJ.

In the first example, the adjective beautiful describes the noun woman. In the second example,
the adjective beautiful describes the pronoun she.

Adverb do three different things. They describe verbs, adjectives, or other adverbs.

She sings beautifully.


VERB ADV.

She is a beautifully dressed woman.


ADV. ADJ. NOUN

She is a truly beautifully dressed woman.


ADV. ADV. ADJ. NOUN

In the first example, the adverb beautifully describes the verb sings. In the second example, the
adverb beautifully describes the adjectives dressed (which describes the noun woman). In the
third example, the adverb truly describes the adverb beautifully, which describes the adjective
dressed (which describe; the noun woman).

46 | Test of English Proficiency


The following are examples of incorrect sentences as they might appear on the TOEFL
test.

They were seated at a largely* table.


ADV. NOUN

The child talked quick* to her mother.


VERB ADJ.

We read an extreme* long story.


ADJ. ADJ.

In the first example, the adverb largely is incorrect because the adjective large is needed to
describe the noun table. In the second example, the adjective quick is incorrect because the
adverb quickly is needed to describe the verb talked. In the last example, the adjective extreme is
incorrect because the adverb extremely is needed to describe the adjective long.

The following chart outlines the important information that you should remember about
the basic use of adjectives and adverbs.

BASIC USE OF ADJECTIVES AND ADVERBS


ADJECTIVES Adjectives describe nouns or pronouns.
ADVERBS Adverbs describe verbs, adjectives, or other adverbs.

SKILL 30: USE ADJECTIVES AFTER LINKING VERBS

Generally an adverb rather than an adjective will come directly after a verb because the adverb is
describing the verb.

She spoke nicely.


VERB ADV.

In the example, the verb spoke is followed by the adverb nicely. This adverb describes the verb
spoke.

However, you must be very careful if the verb is a linking verb. A linking verb is
followed by an adjective rather than an adverb.

She looks nice.


SUB. ADJ.

Test of English Proficiency | 47


In this example, the linking verb looks is followed by the adjective nice. This adjective describes
the subject she.

You should be sure to use an adjective rather than an adverb after a linking verb. Be
careful, however, because the adjective that goes with the linking verb does not always directly
follow the linking verb

He seems unusually nice.


SUB ADV. ADJ.

In the example, the adjective nice, which describes the subject he, is itself described by the
adverb unusually. From this example, you should notice that it is possible to have an adverb
directly after a linking verb, but only if the adverb describes an adjective that follows.

The following chart lists commonly used linking verbs and outlines the different uses of
adjectives and adverbs after regular verbs and linking verbs:

ADJECTIVES AND ADVERBS AFTER VERBS

(subject) + (regular verb) + (adverb)


A regular verb is followed by an adverb. The adverb describes the verb.

(subject) + (linking verb) + (adjective)


A linking verb is followed by an adjective. The adjective describes the subject.

(subject) + (linking verb) + (adverb) + (adjective)


It is possible that a linking verb is followed by an adverb and an adjective. The adverb
describes the adjective, and the adjective describes the subject.
LINKING VERBS: appear feel seem
be look smell
become prove taste

48 | Test of English Proficiency


SKILL 31: POSITION ADJECTIVES AND ADVERBS CORRECTLY

Adjectives and adverbs can appear in incorrect positions in written expression questions on the
TOEFL test. There are two common errors of this type that you should beware of: (1) the
position of adjectives with the nouns they describe, and (2) the position of adverbs with objects.

In English it is correct to place a one-word adjective in front of the noun it describes. On


the TOEFL test, however, an incorrect sentence might have an adjective after the noun it
describes.

The information important* is on the first page.


NOUN ADJ.

In this example, the adjective important should come before the noun information because
important describes information.

A second problem you should be aware of is the position of adverbs with objects of
verbs. When a verb has an object, and adverb describing the verb describing the verb should not
come between the verb and its object.

He has taken recently* an English course.


ADV. OBJECT

The example is incorrect because the adverb recently comes between the verb has taken and its
object an English course. There are many possible corrections for this sentence.

Recently he has taken an English course.

He has recently taken an English course.

He has taken an English course recently.

You can see from these examples that there are many possible correct positions for the adverb.
What is important for you to remember is that an adverb that describes a verb cannot come
between the verb and its object.

The following chart outlines the key points that you should remember about the position
of adjectives and adverbs:

THE POSITION OF ADJECTIVES AND ADVERBS


ADJECTIVES A one-word adjective comes before the noun it describes. It does not
come directly after.
ADVERBS An adverb can appear in many positions. It cannot be used between a
verb and its object.

Test of English Proficiency | 49


Exercise skill 29-31

1. Modern art is on display at the Guggenheim Museum, a building with an unusually


A B C D
design.

2. By the beginning of the 1980s fifteen states had adopted already no-fault insurance laws.
A B C D
3. Heart attacks are fatally in 75 percent of occurences.
A B C D

50 | Test of English Proficiency


SECTION 2
STRUCTURE AND WRITTEN EXPRESSION

Time- 25 minutes
(including the reading of the directions for each part)
Now set your clock for 25 minutes
This section is designed to measure your ability to recognize language that is appropriate for
standard written English. There are two types of questions in this section, with special directions
for each type.

Structure
Directions: Questions 1-15 are incomplete sentences. Beneath each sentence you will see four
word or phrases, marked (A), (B), (C), and (D). Choose the one word or phrase that best
completes the sentence. Then, on your answer sheet, find the number of question and fill in the
space that corresponds to the letter of the answer you have chosen. Fill in the space so that the
letter inside the oval cannot be seen.
Look at the following examples.

Example I
The president the election by a landslide.
(A) Won
(B) He won
(C) Yesterday
(D) Fortunately
The sentence should read, “The president won the election by a landslide.” Therefore, you
should choose (A).

Example II
When the conference?
(A) The doctor attended
(B) Did the doctor attend
(C) The doctor will attend
(D) The doctor’s attendance
The sentence should read, “When did the doctor attend the conference.” Therefore, you should
choose (B).

Test of English Proficiency | 51


Now begin work on the questions.
1. The North Pole a latitude of 90 degrees north.
(A) it has
(B) is having
(C) which is having
(D) has

2. The city of Beverly Hills is surrounded on the city of Los Angeles.


(A) its sides
(B) the sides are
(C) it is the side of
(D) all sides by

3. greyhound, can achieve speeds up to thirty-six miles per hour.


(A) The
(B) The fastest
(C) The fastest dog
(D) The fastest dog, the

4. Marmots spend their time foraging among meadow plants and flowers or on rocky
cliffs.
(A) gets sun
(B) sunning
(C) the sun
(D) sunny

5. The greenhouse effect occurs heat radiated from the Sun.


(A) when does the Earth’s atmosphere trap
(B) does the Earth’s atmosphere trap
(C) when the Earth’s atmosphere traps
(D) the Earth’s atmosphere traps

6. The Rose Bowl place on New Year’s Day, is the oldest postseason collegiate
football game in the United States.
(A) takes
(B) it takes
(C) which takes
(D) took

7. Experiments represent a giant step into the medicine of the future.


(A) using gene therapy
(B) use gene therapy
(C) they use
(D) gene therapy use

52 | Test of English Proficiency


8. off the Hawaiian coastline are living, others are dead.
(A) While some types of coral reefs
(B) Some types of coral reefs
(C) There are many types of coral reefs
(D) Coral reefs

9. Nimbostratus clouds are thick, dark grey clouds forebode rain.


(A) what
(B) which
(C) what they
(D) which they

10. Some economists now suggest that home equity loans are merely a new trap to push
consumers beyond .
(A) they can afford
(B) they can afford it
(C) what is affordable
(D) able to afford

11. People who reverse the letters of words to read suffer from dyslexia.
(A) when trying
(B) if they tried
(C) when tried
(D) if he tries

12. Featured at the Henry Ford Museum of an antique cars dating from 1865.
(A) is an exhibit
(B) an exhibit
(C) an exhibit is
(D) which is an exhibit

13. Rubber from vulcanized silicones with a high molecular weight is difficult to
distinguish from natural rubber.
(A) is produced
(B) producing
(C) that produces
(D) produced

14. appears considerably larger at the horizon than it does overhead is merely an
optical illusion.
(A) The moon
(B) That the moon
(C) When the moon
(D) The moon which

Test of English Proficiency | 53


15. According to the World Health Organization, any of the six most dangerous
diseases to break out, it could be cause for quarantine.
(A) were
(B) they were
(C) there were
(D) were they

Written Expression
Directions: In questions 16-40, each sentence has four underlined words or phrases. the four
underlined parts of the sentence are marked (A), (B), (C), and (D). Identify the one underlined
word or phrase that must be changed in order for the sentence to be correct. Then, on your
answer sheet, find the number of question and fill in the space that corresponds to the letter of
the answer you have chosen. Fill in the space so that the letter inside the oval cannot be seen.
Look at the following examples.

Example I
The four string on a violin are tuned
A B C D
The sentence should read, “The four strings on a violin are tuned.” Therefore, you should choose
(B).

Example II
The research for the book Roots taking Alex Haley twelve years.
A B C D

The sentence should read, “The research for the book Roots took Alex Haley twelve years.”
Therefore, you should choose (C).
Now begin work on the questions.

16. On the floor of the Pacific Ocean is hundreds of flat-topped mountains


A B C
more than a mile beneath sea level.
D
17. Because of the flourish with which John Hancock signed the Declaration of
A
Independence , his name become synonymous with signature.
B C D
18. Segregation in public schools was declare unconstitutional by the Supreme
A B C
Court in 1954.
D

54 | Test of English Proficiency


19. Sirius, the Dog Star, is the most brightest star in the sky with an absolute
A B
magnitude about twenty-three times that of the Sun.
C D
20. Killer whales tend to wander in family clusters that hunt, play, and resting
A B C
together.
D
21. Some of the most useful resistor material are carbon, metals, and metallic alloys.
A B C D
22. The community of Bethesda, Maryland, was previous known as Darcy’s Store.
A B C D
23. Alloys of gold and chopper have been widely using in various types of coins.
A B C D
24. J. H. Pratt used group therapy early in this century when he brought tuberculosis
A B C
patients together to discuss its disease.
D
25. The United States has import all carpet wools in recent years because domestic
A B C
wools are too fine and soft for carpets.
D
26. Irving Berlin wrote “Oh How I Hate to Get Up in the Morning” while serving in
A B
a U.S Army during World War I.
C D
27. Banks are rushing to merge because consolidations enable them to slash theirs
A B C
costs and expand.
D
28. That water has a very high specific heat means that without a large temperature
A B
change water can add or lose a large number of heat.
C D
29. Benny Goodman was equally talented as both jazz performer as well as
A B C
classical musician.
D

Test of English Proficiency | 55


30. The state seal still used in Massachusetts designed by Paul Revere, who also
A B C
designed the first Continental currency.
D
31. Quarter horses were developed in eighteenth-century Virginia to race on
A
courses short of about a quarter of a mile in length.
B C D
32. No longer satisfied with the emphasis of the Denishawn School, Martha Graham
A B C
has moved to the staff of the Eastman School in 1925.
D
33. William Hart was an act best known for his roles as western heroes in silent
A B C D
films.

34. Prior to an extermination program earlier this century, alive wolves roamed
A B C
across nearly all of North America.
D
35. During the 1960s the Berkeley campus of the University of California came to
A B
national attention as a result its radical political activity.
C D
36. Artist Gutzon Borglum designed the Mount Rushmore Memorial and worked on
A
project from 1925 until his death in 1941.
B C D
37. It is proving less costly and more profitably for drugmakers to market directly to
A B C D
patients.

38. Sapphires weighing as much as two pounds have on occasion mined.


A B C D
39. Like snakes, lizards can be found on all others continents except Antartica.
A B C D
40. Banks, saving and loans, and finance companies have recently been doing home
A B
equity loans with greater frequency than ever before.
C D

56 | Test of English Proficiency


READING

MAIN IDEA QUESTIONS


HOW TO IDENTIFY THE On both paper and computer What is the topic of the
QUESTION tests passage?
What is the subject of the
passage?
What is the main idea of the
passage?
What is the author’s main
point in the passage?
What is the author primarily
concerned?
Which of the following would
be the best title?
WHERE TO FIND THE The answer to this type of question can generally be determined
ANSWER by looking at the first sentence of each paragraph
HOW TO ANSWER THE 1. Read the first line of each paragraph
QUESTION 2. Look for a common theme or idea in the first lines
3. Pass your eyes quickly over the rest of the passage that
you have really found the topic sentence(s)
4. Eliminate any definitely wrong answers and choose the
best answer from the remaining choices.

STATED DETAIL QUESTION


HOW TO IDENTIFY THE On both paper and computer According to the passage….
QUESTION tests It is stated in the passage
The passage indicates that
Which of the following is
true…?
WHERE TO FIND THE The answers to these questions are generally found in order in
ANSWER the passage
HOW TO ANSWER THE 1. Choose a key word in the question.
QUESTION 2. Skim in the appropriate part of the passage for the key
word or idea.
3. Read the sentence that contains the key word or idea
carefully.
4. Eliminate the definitely wrong answers and choose the
best answer from the remaining choices.

Test of English Proficiency | 57


“UNSTATED” DETAIL QUESTIONS
HOW TO IDENTIFY THE On both paper and computer Which of the following is not
QUESTION tests stated…?
Which of the following is not
mentioned…?
Which of the following is not
discussed…?
All of the following are true
except…
WHERE TO FIND THE The answers to these questions are generally found in order in
ANSWER the passage
HOW TO ANSWER THE 1. Choose a key word in the question.
QUESTION 2. Scan the appropriate place in the passage for the key
word (or related idea).
3. Read the sentence that contains the key word or idea
carefully.
4. Look for answers that are definitely true according to
the passage. Eliminate those answers.
5. Choose the answer that is not true or not discussed in
the passage.

IMPLIED DETAIL QUESTIONS


HOW TO IDENTIFY THE On both paper and computer It is implied in the passage
QUESTION tests that….
It can be inferred from the
passage that…
It is most likely that…
What probably happened
WHERE TO FIND THE The answers to these questions are generally found in order in
ANSWER the passage
HOW TO ANSWER THE 1. Choose a key word in the question.
QUESTION 2. Scan the passage for the ke word (or a related idea)
3. Carefully read the sentence that contains the key word.
4. Look for an answer that could be true, according to that
sentence.

58 | Test of English Proficiency


VOCABULARY IN CONTEXT QUESTIONS
HOW TO IDENTIFY THE On both paper and computer What is the meaning f “X” in
QUESTION tests line Y?
The word “X” in line Y could
best be replaced by…
WHERE TO FIND THE Information to help you understand the meaning of a
ANSWER vocabulary word can often be found in the context surrounding
the word.
HOW TO ANSWER THE 1. Find the word in the passage.
QUESTION 2. Read the sentence that contains the word carefully.
3. Look for the context clues to help you understand the
meaning.
Choose the answer that the context indicates.

QUESTIONS ABOUT
WHERE IN THE PASSAGE
HOW TO IDENTIFY THE On paper test only Where in the passage …?
QUESTION
WHERE TO FIND THE On paper test only The answer can be in any of
ANSWER the lines listed in the answers
to the question.
HOW TO ANSWER THE 1. Choose a key word or idea in the question.
QUESTION 2. Skim the appropriate part(s) of the passage looking for
the key word or idea.
3. Choose the answer that contains the keyword or idea.

Exercise

In the 1930s, Chester F. Carlson was working in the patents department of a large electronics
firm in New York City. One of the major problems in his work was the length of time and
expense involved in getting patents copied; patents were lengthy legal documents, and the only
ways to get them copied were to take them to a typist or to take them to a photographer. Either
way the idea of copying patents took a lot of time and cost a lot of money.

He came up with the idea for a machine that would copy documents quickly and efficiently. He
researched the idea in the library and then worked over three-year period on developing a
machine that used a light, an electrostatically charged plate, and powder to duplicate images on
paper. The result of his work was machine that produced first xerographic copy on October 22,
1938. He named the process “Xerox,” which means “dry writing.”

Test of English Proficiency | 59


Carlson felt that he had a good idea, one that would be extremely helpful in the business world.
He tried to sell his idea to a number of large corporations, but they were not terribly interested in
this machine. A few years later he sold the process to a small family-owned company. This small
company grew to a giant Xerox Corporation, and both Carlson and Xerox became rather wealthy
in the process.

1. This passage is mainly about


(A) Carlson’s job in a patent office
(B) How the Xerox machine works
(C) Carlson’s success in business
(D) The development of the Xerox machine
2. Which of the following is NOT mentioned as a problem that Carlson encountered in
getting patents copied?
(A) The time needed for copying
(B) The expense of the copying
(C) The length of patents
(D) Dependability of photographers
3. The word “expense” in line 3 is closest in meaning to
(A) Cost
(B) Difficulty
(C) Legality
(D) Payment
4. The following are components of Carlson’s machine EXCEPT
(A) A light
(B) A charged plate
(C) Powder
(D) A typewriter
5. Carlson most likely began work on the machine in
(A) 1930
(B) 1935
(C) 1938
(D) 1941
6. The passage indicates that the large corporations that Carlson tried to sell his process to
(A) Were family owned
(B) Were nonprofit institutions
(C) Helped to develop the process
(D) Did not want to buy his machine

60 | Test of English Proficiency


7. The word “giant” in line 15 could best be replaced by
(A) Monster
(B) Tiny
(C) Familiar
(D) Huge
8. Where in the passage does the author indicate what a patent is?
(A) Lines 1-2
(B) Line 3
(C) Lines 6-7
(D) Lines 9-10

Test of English Proficiency | 61


SECTION 3
READING COMPREHENSION
Time-55 minutes
(including the reading of the directions)
Now set your clock for 55 minutes.

This section is designed to measure your ability to read and understand short passages similar in
topic and style to those that students are likely to encounter in North American universities and
colleges.
Directions: In this section you will re.d several passages. Each one is followed by a number of
questions about it. You are to choose the one best answer, (A), (B), (C), or (D), to each question.
Then, on your answer sheet, find the number of the question and fill in the space that
corresponds to the letter of the answer you have chosen.
Answer all questions about the information in a passage on the basis of what is stated or implied
in that passage.
Read the following passage:
John Quincy Adams, who served as the sixth president of the United States from 1825 to I 829,
is today recognized for his masterful statesmanship and diplomacy. He dedicated his life to
public service, both in the presidency and in the various other political offices that he held.
Throughout his political career he demonstrated his unswerving belief in freedom of speech, the
antislavery cause and the right of Americans to be free from European and Asian domination.
Example I
To what did John Quincy Adams devote his life?
(A) Improving his personal life
(B) Serving the public
(C) Increasing his fortune
(D) Working on his private business
According to the passage, John Quincy Adams “dedicated his life to public service.” Therefore,
you should choose (B).
Example II
In line 4, the word “unswearing” is closest meaning to

(A) moveable
(B) insignificant
(C) unchanging
(D) diplomatic

62 | Test of English Proficiency


the passage states that John Quincy Adams demonstrated his unswearing belief “throughout his
career.” This implies that he belief did not change. Therefore, you should choose (C).

Now, begin work on the questions.

Questions 1-9

Carbon tetrachloride is a colorless and inflammable liquid that can be produced by


combining carbon disulfide and chlorine. This compound is widely used in industry today
because of its effectiveness as a solvent as well as its use in the production of propellants.

Despite its widespread use in industry, carbon tetrachloride has been banned for home
use. In the past, carbon tetrachloride was a common ingredient in cleaning compounds that were
used throughout the home, but it was found to be dangerous: when heated, it changes in a
poisonous gas that can cause severe illness and even death if it is inhaled. Because of this
dangerous characteristic, throughout the home, but it was found to be dangerous: when heated, it
changes into a poisonous gas that can cause severe illness and even death if it is inhaled. Because
of this dangerous characteristic, the United States revoked permission for the home use of carbon
tetrachloride in 1970. The United States has taken similar action with various other chemical
compounds.

1. The main point of this passage is that


(A) Carbon tetrachloride can be very dangerous when it is heated
(B) The government banned carbon tetrachloride in 1970
(C) Although carbon tetrachloride can legally be used in industry, it is not allowed in home
products
(D) Carbon tetrachloride used to be a regular part of cleaning compounds

2. The word "widely" in line 2 could most easily be replaced by


(A) grandly
(B) extensively
(C) largely
(D) hugely

3. The word "banned" in line 4 is closest in meaning to


(A) forbidden
(B) allowed
(C) suggested
(D) instituted

Test of English Proficiency | 63


4. According to the passage, before 1970 carbon tetrachloride was
(A) used by itself as a cleanser
(B) banned in industrial use
(C) often used as a component of cleaning products
(D) not allowed in home cleaning product

5. It is stated in the passage that when carbon tetrachloride is heated, it becomes


(A) harmful
(B) colorless
(C) a cleaning compound
(D) inflammable

6. The word "inhaled" in line 7 is closest in meaning to


(A) warmed
(B) breathed in
(C) carelessly used
(D) blown

7. The word "revoked" in line 8 could most easily be replaced by


(A) gave
(B) granted
(C) instituted
(D) took away

8. It can be inferred from the passage that the one role of the U.S. government is to
(A) regulate product safety
(B) prohibit any use of carbon tetrachloride
(C) instruct industry on cleaning methodologies
(D) ban the use of any chemicals

9. The paragraph following the passage likely discusses


(A) additional uses for carbon tetrachloride
(B) the banning of various chemical compounds by the U.S. government
(C) further dangerous effects of carbon tetrachloride
(D) the major characteristics of carbon tetrachloride

64 | Test of English Proficiency


Questions 10-19

The next artist in this survey of American artists is James Whistler; he is included in this survey
of American artists because he was born in the United States, although the majority of his
artwork was completed in Europe. Whistler was born in Massachusetts in 1834, but nine years
later his father moved the family to St. Petersburg, Russia, to work on the construction of a
railroad. The family returned to the United States in 1849. Two years later Whistler entered the
U.S. military academy at returned to the West Point, but he was unable to graduate. At the age of
twenty-one, Whistler went to Europe to study art despite familial objections, and he remained in
Europe until his death.

Whistler worked in various art forms, including etchings and lithographs. However, he is most
famous for his paintings, particularly Arrangement in Gray and Black No. 1: Portrait of the
Artist’s Mother or Whistler’s Mother, as it is more commonly known. This painting shows a side
view of Whistler’s mother, dressed in black and posing against a gray wall. The asymmetrical
nature of the portrait, with his mother seated off-center, is highly characteristic of Whistler’s
work.

10. The paragraph preceding this passage most likely discusses


(A) a survey eighteenh-century art
(B) a different American artist
(C) Whistler’s other famous painting
(D) European artists

11. Which of the following best describes the information in the passage?
(A) Several artists are presented.
(B) One artist’s life and works are described.
(C) Various paintings are contrasted.
(D) Whistler’s family life is outlined.

12. Whistler is considered an American artist because


(A) he was born in America
(B) he spent most of his life in America
(C) he served in the U.S. military
(D) he created most of his famous art in America

13. The word “majority” in line 2 is closest in meaning to


(A) seniority
(B) maturity
(C) large pieces
(D) high percentage

Test of English Proficiency | 65


14. It is implied in the passage that Whistler’s family was
(A) unable to find any work at all in Russia
(B) highly supportive of his desire to pursue art
(C) working class
(D) military

15. The word “objections” in line 7 is closest in meaning to


(A) protests
(B) goals
(C) agreements
(D) battles

16. In line 8, the “etchings” are


(A) a type of painting
(B) the same as a lithograph
(C) an art form introduced by Whistler
(D) an art form involving engraving

17. The word “asymmetrical” in line 11 is closest in meaning to


(A) proportionate
(B) uneven
(C) balanced
(D) lyrical

18. Which of the following is NOT true according to the passage?


(A) Whistler worked with a variety of art forms.
(B) Whistler’s Mother is not the official name of his painting.
(C) Whistler is best known for his etchings.
(D) Whistler’s Mother is painted in somber tones.

19. Where in the passage does author mention the types of artwork that Whistler was involved
in?
(A) Lines 1-3
(B) Lines 4-5
(C) Lines 6-7
(D) Lines 8-10

66 | Test of English Proficiency


Questions 20-30

The locations of stars in the sky relative to one another do not appear to the naked eye to change,
and as a result stars are often considered to be fixed in position. Many unaware stargazers falsely
assume that each star has its own permanent home in the nighttime sky.

In reality, though, stars are always moving, but because of the tremendous distances between
stars themselves and from stars to Earth, the changes are barely perceptible here. An example of
a rather fast-moving demonstrates why this misconception prevails; it takes approximately 200
years for a relatively rapid star like Bernard’s star to move distance in the skies equal to the
diameter of the earth’s moon. When the apparently negligible movement of the stars in
contrasted with the movement of the planets, the stars are seemingly unmoving.

20. Which of the following is the best title for this passage?
(A) What the Eye Can See in the Sky
(B) Bernard’s Star
(C) Planetary Movement
(D) The Ever moving Stars

21. The expression “naked eye” in line 1 most probably refers to


(A) a telescope
(B) a scientific method for observing stars
(C) unassisted vision
(D) a camera with a powerful lens

22. According to the passage, the distances between the stars and Earth are
(A) barely perceptible
(B) huge
(C) fixed
(D) moderate

23. The word “perceptible” in line 5 is closest in meaning to which of the following?
(A) Noticeable
(B) Persuasive
(C) Conceivable
(D) Astonishing

24. In line 6, a “misconception” is closest in meaning to a(n)


(A) idea
(B) proven fact
(C) erroneous belief
(D) theory
Test of English Proficiency | 67
25. The passage states that in 200 years Bernard’s star can move
(A) around Earth’s moon
(B) next to Earth’s moon
(C) a distance equal to the distance from Earth to the Moon
(D) a distance seemingly equal to the diameter of the Moon

26. The passage implies that from Earth it appears that the planets
(A) are fixed in the sky
(B) move more slowly than the stars
(C) show approximately the same amount of movement as the stars
(D) travel through the sky considerably more rapidly than the stars

27. The word “negligible” in line 8 could most easily be replaced by


(A) negative
(B) insignificant
(C) rapid
(D) distant

28. Which of the following is NOT true according to the passage?


(A) Starts do not appear to the eye to move.
(B) The large distances between stars and the earth tend to magnify movement to the eye.
(C) Bernard’s star moves quickly in comparison with other stars
(D) Although stars move, they seem to be fixed.

29. The paragraph following the passage most probably discusses


(A) the movement of the of the planets
(B) Bernard’s star
(C) the distance from Earth to the Moon
(D) why stars are always moving

30. This passage would most probably be assigned reading in which course?
(A) Astrology
(B) Geophysics
(C) Astronomy
(D) Geography]

68 | Test of English Proficiency


Questions 31-40

It has been noted that, traditionally, courts have granted divorces on fault grounds: one spouse is
deemed to be at fault in causing the divorce. More and more today, however, divorces are being
granted on a no-fault basis.

Proponents of no-fault divorce argue that when a marriage fails, it is rarely the case that one
marriage partner is completely to blame and the other blameless. A failed marriage is much more
often the result of mistakes by both partners.

Another argument in favor of no-fault divorce is that proving fault in court, in a public arena, is a
destructive process that only serves to lengthen the divorce process and that dramatically
increases the negative feelings present in a divorce. If a couple can reach a decision to divorce
without first deciding which partner is to blame, the divorce settlement can be negotiated more
easily and equitably and the post-divorce healing process can begin more rapidly.

31. What does the passage mainly discuss?


(A) Traditional grounds for divorce
(B) Who is at fault in a divorce
(C) Why no-fault divorces are becoming more common
(D) The various reasons for divorces

32. The word “spouse” in line 1 is closest in meaning to a


(A) judge
(B) problem
(C) divorce decree
(D) marriage partner

33. According to the passage, no-fault divorces


(A) are on the increase
(B) are the traditional form of divorce
(C) are less popular than they used to be
(D) were granted more in the past

34. It is implied in the passage that


(A) there recently has been a decrease in no-fault divorces
(B) not all divorces today are no-fault divorces
(C) a no-fault divorces is not as equitable as a fault divorce
(D) people recover more slowly from a no-fault divorce

Test of English Proficiency | 69


35. The word “Proponents” in line 4 is closest in meaning to which of the following?
(A) Advocates
(B) Recipients
(C) Authorities
(D) Enemies

36. The passage states that a public trial to prove the fault of one spouse can
(A) be satisfying to the wronged spouse
(B) lead to a shorter divorce process
(C) reduce negative feelings
(D) be a harmful process

37. Which of the following is NOT listed in this passage as an argument in favor of no-fault
divorce?
(A) Rarely is only one marriage partner to blame for a divorce
(B) A no-fault divorce generally costs less in legal fees
(C) Finding fault in a divorce increases negative feelings
(D) A no-fault divorce settlement is generally easier to negotiate

38. The word “present” in line 9 could most easily be replaced by


(A) existing
(B) giving
(C) introducing
(D) resulting

39. The word “settlement” in line 10 is closest in meaning to


(A) development
(B) serenity
(C) discussion
(D) agreement

40. The tone of this passage is


(A) emotional
(B) enthusiastic
(C) expository
(D) reactionary

70 | Test of English Proficiency


Questions 41-50

Whereas literature in the first half of the eighteenth century in America had been largely
religious and moral in tone, by the latter half of the century the revolutionary fervor that was
coming to life in the colonies began to be reflected in the literature of the time, which in turn
served to further influence the population. Although not all writers of this period supported the
Revolution, the two best-known and most influential writers, Ben Franklin and Thomas Paine,
were both strongly supportive of that cause.

Ben Franklin first attained popular success through his writings in his brother’s newspaper, the
New England Current. In these articles he used a simple style of language and common sense
argumentation to defend the point of view of the farmer and the Leather Apron man. He
continued with the same common sense practically and appeal to the common man with his work
on Poor Richard’s Almanac from 1733 until 1758. Firmly established in his popular acceptance
by the people Franklin wrote on variety of extremely effective articles and pamphlets about the
colonists’ revolutionary cause against England.

Thomas Paine was an Englishman working as a magazine editor in Philadelphia at the time of
Revolution. His pamphlet Common Sense, which appeared in 1776, was a force in encouraging
the colonists to declare their independence from England. Then throughout the long an desperate
war years, he published a series of Crisis papers (from 1776 until 1783) to encourage the
colonists to continue on struggle. The effectiveness of his writing was probably due to his
emotional yet oversimplified depiction of the cause of the colonists against England as a classic
struggle of good and evil.

41. The paragraph preceding this passage most likely discusses


(A) how literature influences the population
(B) religious and moral literature
(C) literature supporting the cause of the American Revolution
(D) what made Thomas Paine’s literature successful

42. The word “fervor” in line 2 is closest in meaning to


(A) war
(B) anxiety
(C) spirit
(D) action

43. The word “time” in line 3 could best be replaced by


(A) hour
(B) period
(C) appointment
(D) duration

Test of English Proficiency | 71


44. It is implied in the passage that
(A) some writers in the American colonies supported England during the Revolution
(B) Franklin and Paine were the only writers to influence the Revolution
(C) because Thomas Paine was an Englishman, he supported England against the colonies
(D) authors who supported England did not remain in the colonies during the Revolution
45. The pronoun “he” in line 8 refers to
(A) Thomas Paine
(B) Ben Franklin
(C) Ben Franklin’s brother
(D) Poor Richard
46. The expression “point of view” in line 9 could best be replaced by
(A) perspective
(B) sight
(C) circumstance
(D) trait
47. According to the passage, the tone of Poor Richard’s Almanac is
(A) pragmatic
(B) erudite
(C) theoretical
(D) scholarly
48. The word “desperate” in line 16 could best be replaced by
(A) unending
(B) hopeless
(C) strategic
(D) combative
49. Where in the passage does the author describe Thomas Paine’s style of writing?
(A) Lines 4-6
(B) Lines 8-9
(C) Lines 14-15
(D) Lines 18-20
50. The purpose of the passage is to
(A) discuss American literature in the first half of the eighteenth century
(B) give biographical data on two American writers
(C) explain which authors supported the Revolution
(D) describe the literary influence during revolutionary America

72 | Test of English Proficiency

You might also like